3
$\begingroup$

Consider the heat-Schrodinger evolution $e^{(1+i)t\Delta}$, $t\geq 0$. For simplicity, suppose to work in dimension three. Due to the Strichartz estimates for the Schrodinger equation, and the smoothing effect of the heat flow, I expect that the following estimate holds true: \begin{equation}\label{tre} (1)\qquad \left\|\int_0^te^{(1+i)(t-s)\Delta}F(s)ds\right\|_{L^2((0,t),W^{1,6}(\mathbb{R}^3))}\lesssim \|F\|_{L^2((0,t),L^2(\mathbb{R}^3))} \end{equation} Nevertheless, I have not been able to prove it.

Is estimate (1) actually true? In case, is it already known in the literature?

Thank you for your suggestions.

$\endgroup$
1
  • $\begingroup$ Sure, thank you. I edited $\endgroup$
    – Capublanca
    Mar 4, 2020 at 1:12

1 Answer 1

4
$\begingroup$

I think the estimate you want does not really require Strichartz.

First, your estimate is equivalent to the following, which is a bit easier for me to think about: let $u$ be the solution to the equation $$ \partial_t u - (1 + i) \triangle u = F \tag{*}$$ with initial data $$ u(0,x) \equiv 0 $$ Then the desired estimate is $$ \| u \|_{L^2_t W^{1,6}_x} \leq \|F\|_{L^2_t L^2_x}$$

We can derive this estimate by multiplying the equation (*) by $\triangle \bar{u}$ and taking the real part, which gives $$ \partial_t u \triangle \bar{u} + \partial_t \bar{u} \triangle u - (1+i) \triangle u \triangle \bar{u} - (1-i) \triangle u \triangle\bar{u} = F \triangle \bar{u} + \bar{F} \triangle u $$ Integrate by parts in space we get $$ \partial_t \| \nabla u\|^2_{L^2_x} + 2 \|\triangle u\|_{L^2_x}^2 = - \int_{\mathbb{R}^3} F\triangle \bar{u} + \bar{F} \triangle u ~dx$$ Integrate between time 0 and time T you get (using the triviality of the data) $$ \|\nabla u(T)\|_{L^2_x}^2 + 2 \int_0^T \| \triangle u\|^2_{L^2_x} ~dt \leq \int_{[0,T]\times \mathbb{R}^3} 2 |F|\cdot |\triangle u| ~dx~dt (**)$$ Applying Young's inequality on the right, you can pull out $\epsilon \|\triangle u\|_{L^2_t L^2_x}$ which can be absorbed on the left hand side, and this reduces to

$$ \| \triangle u\|_{L^2_t L^2_x} \lesssim \| F\|_{L^2_t L^2_x} $$

Finally use elliptic estimates to bound $\|\nabla^2 u\|$ by $\|\triangle u\|$, and then Sobolev inequality in space gets you

$$ \| \nabla u\|_{L^2_t L^6_x} \lesssim \|F\|_{L^2_t L^2_x} $$

This takes care the homogeneous part of the Sobolev norm.

I don't think the inhomogeneous part holds. It doesn't have the correct scaling.

If you rescale $u(t,x) \mapsto u(\lambda^2 t, \lambda x)$ by the parabolic scaling, then $F(t,x) \mapsto \lambda^2 F(\lambda^2 t, \lambda x)$. The $L^2_t L^2_x$ norm of $F$ scales like $\lambda^{-1/2}$, but the $L^2_t L^6_x$ norm of $u$ scales like $\lambda^{-3/2}$. The term with the correct scaling should be $\|F\|_{L^1_t L^2_x}$. And the estimate

$$ \| u\|_{L^2_t L^6_x} \lesssim \|\nabla u\|_{L^2_t L^2_x} \lesssim \|F\|_{L^1_t L^2_x} $$

can be proven by an analogous manner as above but instead of multiplying by $\triangle u$, multiply by $u$.

Incidentally: both estimates also hold for the heat equation, without the Schroedinger part.


Addendum: I just noticed that your inequality is only stated for $L^2((0,T),X)$ for some Banach space $X$. Do you only care about the local estimate where your constant can depend on the length of the interval? Or do you actually want uniform estimates for all $T$? I ask because quite obviously on a fixed time interval you have that $\|F\|_{L^1_t L^2_x} \lesssim \|F\|_{L^2_t L^2_x}$, and in fact you can get the same result by noting that (**) also implies $$\|u\|_{L^\infty_t L^6_x} \lesssim \|F\|_{L^2_t L^2_x}$$ and can be easily localized to intervals.

$\endgroup$
5
  • $\begingroup$ Nice. I think for the lower order term you might need some weight at the RHS $\endgroup$ Mar 4, 2020 at 8:31
  • $\begingroup$ @PieroD'Ancona: I don't quite understand what you mean. Can you say a couple sentences more? What I imagine: multiplying by $u$ and IBP gives us that $$\|u(T)\|_{L^2}^2 + \| \nabla u\|_{L^2_{t,x}}^2 \leq \| F u \|_{L^1_{t,x}} $$ the right hand side bound is bounded by $\| F\|_{L^1_t L^2_x} \|u\|_{L^\infty_t L^2_x}$ which implies $$ \|u\|_{L^\infty_t L^2_x} \leq \|F\|_{L^1_t L^2_x}$$ which gives the desired result. I don't think weights are necessary. But possibly I misunderstand your comment. $\endgroup$ Mar 4, 2020 at 14:13
  • $\begingroup$ What I mean is that another way to achieve the correct scaling is, say, using a weighted norm such as $\||x|F\|_{L^2L^2}$. I think the estimate for the lower order term might be true also with this right hand side $\endgroup$ Mar 4, 2020 at 14:58
  • $\begingroup$ @PieroD'Ancona: ah, I see. For the pure Schrodinger this would be an end-point case if I am not mistaken. (It almost follows from my paper arxiv.org/abs/1701.01460v4 but it would be the endpoint case which I didn't do). $\endgroup$ Mar 4, 2020 at 16:21
  • $\begingroup$ Nice work, thank you. I was indeed interested only in the homogeneous estimates, but also the suggestion by Piero for the lower order term is very interesting. I think that, if one considers the evolution $e^{(i+\varepsilon)t\Delta}$, where $\varepsilon>0$ is a small parameter, your approach produces a divergence rate as $\varepsilon\to 0$ which can be improved by using the Strichartz estimates for the Schrodinger equation. Maybe it could be interesting (and not so trivial) to establish the optimal divergence rate as $\varepsilon\to 0$. $\endgroup$
    – Capublanca
    Mar 4, 2020 at 19:29

Your Answer

By clicking “Post Your Answer”, you agree to our terms of service and acknowledge you have read our privacy policy.

Not the answer you're looking for? Browse other questions tagged or ask your own question.